Fix typos in Serway and Jewett v8's 23.62 and 25.21.
authorW. Trevor King <wking@tremily.us>
Fri, 20 Apr 2012 21:07:16 +0000 (17:07 -0400)
committerW. Trevor King <wking@tremily.us>
Fri, 20 Apr 2012 21:07:16 +0000 (17:07 -0400)
latex/problems/Serway_and_Jewett_8/problem23.62.tex
latex/problems/Serway_and_Jewett_8/problem25.21.tex

index 6c9dc063283b367fb85fc97d79a6a15ea02f8539..eb338d3efeeb8a8e68ccf8df3c47e55c7c35a2d6 100644 (file)
@@ -1,6 +1,6 @@
 \begin{problem*}{23.62}
 Four identical charged particles ($q=+10.0\U{$\mu$C}$) are located on
-the corners of a rectangle as shown in Figrue P23.62.  The dimensions
+the corners of a rectangle as shown in Figure P23.62.  The dimensions
 of the rectangle are $L=60.0\U{cm}$ and $W=15.0\U{cm}$.
 Calculate \Part{a} the magnitude and \Part{b} the direction of the
 total electric force exerted on the charge at the lower left corner by
index 403a08fc34ef37036265b89e039ec93e7778e871..116769bb2850cc55467c4a06644a3e3645333153 100644 (file)
@@ -1,10 +1,10 @@
 \begin{problem*}{25.21}
-Two particles each with charge $+2.00\U{$\mu$C}$ are lopcated on the
+Two particles each with charge $+2.00\U{$\mu$C}$ are located on the
 $x$ axis.  One is at $x=1.00\U{m}$, and the other is at
 $x=-1.00\U{m}$.  \Part{a} Determine the electric potential on the $y$
 axis at $y=0.500\U{m}$.  \Part{b} Calculate the change in electric
 potential energy of the system as a third charged particle of
-$-3.00\U{$\mu$C}$ is brought in from infinitely far away to a positino
+$-3.00\U{$\mu$C}$ is brought in from infinitely far away to a position
 on the $y$ axis at $y=0.500\U{m}$.
 \end{problem*}